Discussion

The conclusion of the argument follows logically if which one of the following is assumed?
(A)No celestial body can simultaneously be a moon and a planet.
(B)...
(C)...
(D)...
(E)...
(F)...
*This question is included in June 2012 LSAT (PT66): Logical Reasoning B, question #17

The solution is

Posted: 01/05/2013 19:25
Why is C incorrect? Many of these answers could be correct...
Posted: 02/01/2013 19:55
Because perhaps a body which used to be a moon could still be considered a current planet, even if different circumstances would have led to it remaining a moon.
Image Not Available
Contributor
Posted: 02/01/2013 19:59
Want to ace the LSAT? Check out our program. Visit CollegeReview.Org.

You need to be signed in to perform that action.

Sign In